subject
Computers and Technology, 25.07.2019 04:00 xojade

3. if b3=6 and d5=8, what would the following function return? if(b3> d5, "closed", d5-b3) * a. "closed" b. -2 c. "open" d. +2

ansver
Answers: 2

Other questions on the subject: Computers and Technology

image
Computers and Technology, 21.06.2019 19:10, gummybear1398
Select the correct answer. emma, the quality control manager in a software development company, asks the testing team to check whether the user interface is friendly. to which type of testing is emma referring? a. usability testing b. data comparison c. validation testing d. stress testing e. destruction testing
Answers: 1
image
Computers and Technology, 22.06.2019 11:30, neekWYB
Andrina writes letters that are regularly sent to hundreds of her company’s customers. because of this, she would like for the mail merge command to be in her quick access toolbar, and she wants it to be the first button on the left. what should andrina do to place the mail merge button there?
Answers: 1
image
Computers and Technology, 22.06.2019 20:00, ksanchez2100
Need asap assignment directions: think of an organization (business, religious institution, volunteer organization, sports team) with which you have been involved. imagine outfitting it with an it infrastructure. prepare a plan for what you would do to support outfitting it. draw a map of a network connecting all the individuals, give them pcs and printers, and lay out the design as best you can. the purpose is to begin working with these concepts, not to build a perfect network.
Answers: 2
image
Computers and Technology, 22.06.2019 23:50, Emptypockets451
You need to design a circuit that implements the functions in the following table: s0 s1 function0 0 a + 10 1 a – b1 0 a + b1 1 a – 1s0 and s1 are 1-bit control inputs to select the function of the circuit. inputs a and b are 4-bitnumbers in 2s complement form. the output is also a 4-bit number in 2s complement form. you are allowed to use only one ttl 7483 4-bit adder to implement all the functions. but anynumber of other components (except the adder) can be used. hint: design a combinational logic circuit to modify the input b and the “carry input” of theadder depending on the control inputs s0 and s1.important: lab grade will depend on the working of the circuit & will be checked of by your labinstructor.1. is the output valid for the following input combinations: a. s0 = 0, s1 = 0, a = 7, b = 3? b. s0 = 0, s1 = 1, a = 7, b = 3? c. s0 = 1, s1 = 0, a = -4, b = -5? d. s0 = 1, s1 = 1, a = -8, b = 6? 2. what is the range of inputs (for both a and b) that will produce the valid output for all the functions?
Answers: 3
You know the right answer?
3. if b3=6 and d5=8, what would the following function return? if(b3> d5, "closed", d5-b3) * a....

Questions in other subjects:

Konu
Mathematics, 26.10.2020 22:50
Konu
Mathematics, 26.10.2020 22:50